LSAT and Law School Admissions Forum

Get expert LSAT preparation and law school admissions advice from PowerScore Test Preparation.

 Administrator
PowerScore Staff
  • PowerScore Staff
  • Posts: 8917
  • Joined: Feb 02, 2011
|
#22865
Complete Question Explanation

Main Point. The correct answer choice is (B)

The author concludes that cutting City Zoo's funding in half would be false economy. Three reasons are offered in support of this conclusion: first, the zoo's budget is too small to have any impact on the city's finances; second, cutting the budget will force the zoo to close and the city will lose the tax and tourist dollars that it generates; third, the city will lose a cultural icon that makes it an attractive place for business. Since this is a Main Point question, all you need to do is retain a clear grasp of the conclusion of the argument.

Answer choice (A): While the city must reduce spending (first sentence), this is not the main conclusion of the argument, which is about not cutting the zoo's funding in half.

Answer choice (B): This is the correct answer choice. This answer is a direct paraphrase of the second sentence of the stimulus.

Answer choice (C): While the stimulus contains sufficient evidence to support the idea that City Zoo's budget is only a small portion of the city's entire budget, this is not the conclusion of the argument. Rather, we are dealing with a premise that is used in support of the conclusion. Since we are not answering a Must Be True question, this answer choice is incorrect.

Answer choice (D): As with answer choice (C), the stimulus contains sufficient evidence to support the inference presented here. However, the unfortunate impact of budget cuts upon the zoo only gives a reason why the zoo's funding should not be cut; it is not the main point of the argument. Since we are not answering a Must Be True question, this answer choice is incorrect.

Answer choice (E): This answer choice is an exaggeration, since we are given no information about how the zoo's closure will impact the city's educational climate. Even if such damage were irreparable, this would only support the main conclusion of the argument. This answer choice is incorrect.
 nickp18
  • Posts: 20
  • Joined: May 26, 2020
|
#95515
Hello!

Quick question to help confirm my reasoning! I selected the correct answer, but was going back and forth between A and B. I ultimately eliminated A because I thought it was a sub-conclusion and because "reducing spending is the ONLY means" is not only extreme, but also not stated anywhere else.


Thank you!
 Robert Carroll
PowerScore Staff
  • PowerScore Staff
  • Posts: 1787
  • Joined: Dec 06, 2013
|
#95546
nickp18,

In fact the first sentence of the stimulus does say that the city must reduce its spending. "Must" is as much a necessary condition indicator as "only". The problem is that the author offers no evidence that the first sentence is true, so it's not a conclusion - not the main conclusion and not a subconclusion.

Robert Carroll

Get the most out of your LSAT Prep Plus subscription.

Analyze and track your performance with our Testing and Analytics Package.